Notifiche
Cancella tutti

[Risolto] TEOREMA DI DE L'HOPITAL

  

1
569
Autore
1 Risposta



1

a.   

$\displaystyle\lim_{x \to 0^-} \frac {e^{-\frac{1}{x}}}{ln(-x)}$

Forma indeterminata del tipo ∞/∞

Usiamo de l'Hôpital 

$\displaystyle\lim_{x \to 0^-} \frac {\frac {e^{-\frac{1}{x}}}{x^2}}{\frac{1}{x}}$

$\displaystyle\lim_{x \to 0^-} \frac {e^{-\frac{1}{x}}}{x} = -\infty$

nota. Il numeratore → +∞ mentre il denominatore → 0⁻.

 .

b.  

$\displaystyle\lim_{x \to 0^+} \frac {e^{-\frac{1}{x}}}{ln(x)}$

Non è una forma indeterminata.

i) il numeratore → 0

ii) il denominatore → -∞ 

$\displaystyle\lim_{x \to 0^+} \frac {e^{-\frac{1}{x}}}{ln(x)} = 0$



Risposta
SOS Matematica

4.6
SCARICA